Greit polynom

Her kan brukere av forum utfordre hverandre med morsomme oppgaver og nøtter man ønsker å dele med andre. Dette er altså ikke et sted for desperate skrik om hjelp, de kan man poste i de andre forumene, men et sted for problemløsing på tvers av trinn og fag.

Moderatorer: Vektormannen, espen180, Aleks855, Solar Plexsus, Gustav, Nebuchadnezzar, Janhaa

Svar
Markus
Fermat
Fermat
Innlegg: 767
Registrert: 20/09-2016 13:48
Sted: NTNU

La $P$ være et polynom med heltallskoeffisienter. Hvor mange (forskjellige) polynom finnes det slik at $P(a)=b$, $P(b)=c$ og $P(c)=a$? der $a,b,c$ er distinkte heltall?
Aleks855
Rasch
Rasch
Innlegg: 6855
Registrert: 19/03-2011 15:19
Sted: Trondheim
Kontakt:

Gitt kriteriene får vi for eksempel at $P(c) = P(P(b)) = P(P(P(a))) = a$. Det lukter at dette skal lede et sted, men jeg kommer ikke videre.

En annen sti:

$$P(a) = b \wedge P(b) = c \Rightarrow P(a) - P(b) = b-c \Rightarrow \color{blue}{b-c = k(a-b)} \ \text{for en} \ k \in \mathbb Z$$
$$P(b) = c \wedge P(c) = a \Rightarrow P(b) - P(c) = c-a \Rightarrow \color{blue}{c-a = m(b-c)} \ \text{for en} \ m \in \mathbb Z$$
$$P(c) = a \wedge P(a) = b \Rightarrow P(c) - P(a) = a-b \Rightarrow \color{blue}{a-b = n(c-a)} \ \text{for en} \ n \in \mathbb Z$$

Ganger vi sammen disse resultatene får vi at $(a-b)(b-c)(c-a) = kmn(a-b)(b-c)(c-a)$.

Dersom $kmn = 1$ må $k=m=n=1$, som betyr at $a=b=c$ som er en selvmotsigelse.
Dersom $kmn \neq 1$ bryter siste likning sammen trivielt, og er også en selvmotsigelse.

Dette ble litt mer rotete enn jeg forestilte meg, men holder det?
Bilde
Markus
Fermat
Fermat
Innlegg: 767
Registrert: 20/09-2016 13:48
Sted: NTNU

Ser bra ut det der, også ganske bra ført! Selv om det kanskje er trivielt, så ville jeg kanskje tatt med at hvis $P$ er et polynom med heltallskoeffisienter og $a,b$ heltall, så $(a-b) \mid (P(a)-P(b))$, da jeg regner med at du har brukt det. Det er riktignok ikke riktig at $k=m=n=1$, vi kan også ha $k,m=\pm 1$, $n=\mp 1$. Jeg tror du allikevel skal klare å få den samme selvmotsigelsen.

En alternativ måte å avslutte beviset på er at siden $(a-b) \mid (b-c)$, $(b-c) \mid (c-a)$ og $(c-a) \mid (a-b)$, så vil $(a-b) \mid (b-c) \mid (c-a) \mid (a-b)$. Da må $(b-c)=\pm (a-b)$ og $(c-a)=\pm (a-b)$, som betyr at $a=b$ som gir selvmotsigelsen.
Svar